Teorías de calibre de monopolos magnéticos

Estoy citando a 't Hooft:

"[...] Pueden existir configuraciones de campo localmente estables que tengan algún giro topológico en ellas [...]. Análisis cuidadoso de los grupos de Lie existentes y la forma en que pueden dividirse espontáneamente en uno o más subgrupos tu ( 1 ) , revela una característica general: solo si el grupo de calibre subyacente es compacto y tiene un grupo de cobertura compacto, las cargas eléctricas en el tu ( 1 ) cuantizar los grupos de calibre (de lo contrario, no estaría prohibido agregar números reales arbitrarios a los tu ( 1 ) cargas), y siempre que el grupo de cobertura del grupo de calibre subyacente sea compacto, se pueden construir soluciones de monopolo magnético. [...]"

  1. ¿Cuáles son los grupos de cobertura?

  2. ¿Qué quiso decir con que las cargas eléctricas se cuantifican solo cuando el grupo de calibre y el grupo de cobertura son compactos?

  3. Y finalmente, ¿cómo se pueden construir monopolos magnéticos a partir de cargas eléctricas cuantificadas?

Respuestas (1)

1) Los grupos de cobertura universal son grupos con la propiedad de estar simplemente conectados. Cada álgebra tiene un grupo de cobertura único. Los otros grupos, { GRAMO } , asociado a la misma álgebra se puede obtener del grupo de cobertura de la siguiente forma

GRAMO = GRAMO ~ k mi r ( ρ ) ,
dónde k mi r ( ρ ) es el núcleo del homomorfismo de grupo ρ : GRAMO ~ GRAMO . Una vez que haya definido una representación particular, podrá calcular este kernel. Por ejemplo, comienzas con un s tu ( 2 ) álgebra. Entonces, si elige la representación adjunta, puede mostrar que k mi r ( ρ ) = Z 2 y el grupo sera GRAMO = S tu ( 2 ) / Z 2 = S O ( 3 ) . Por otro lado, si eliges la representación definitoria obtienes k mi r ( ρ ) = 1 y GRAMO = S tu ( 2 ) / 1 = S tu ( 2 ) .

2) Un monopolo magnético topológico tiene que satisfacer la condición de cuantificación

mi i mi q metro = 1 ,
dónde q metro es la carga magnética (no abeliana). Esta es una generalización de la condición de cuantificación de Dirac . Se puede demostrar que para satisfacer esta condición el tu ( 1 ) tiene que ser compacto porque la carga eléctrica también tiene que ser cuantificada. No estoy muy seguro del resultado que está afirmando: "un compacto GRAMO con revestimiento compacto GRAMO ~ implica en tu ( 1 ) compacto". El resultado que sé es que cuando tienes una ruptura de simetría espontánea GRAMO k × tu ( 1 ) , el tu ( 1 ) es compacto si GRAMO y k ambos son semisimples . De lo contrario tu ( 1 ) puede ser no compacto.

3) Los monopolos magnéticos no se construyen con cargas eléctricas. Sin embargo, se obtienen en teorías de gauge quebradas espontáneamente que, en general, tienen cargas eléctricas en su espectro. Supongo que solo quiso decir que las cargas eléctricas cuantificadas implican cargas magnéticas cuantificadas y viceversa.